Why does trying to compute $lim_{xto-infty} {2x-1over sqrt{3x^2+x+1}}$ result in the negative of the answer...












24












$begingroup$


My textbook asks me to evaluate the limit $$lim_{xto-infty} {2x-1over sqrt{3x^2+x+1}}$$ which evaluates to $-2oversqrt{3}$. The method in the book is to factor out $x^2$ from the root in the denominator:



$$begin{align}
lim_{xto-infty} {2x-1over sqrt{3x^2+x+1}} & = lim_{xto-infty} {2x-1over sqrt{x^2left(3+frac{1}{x}+frac{1}{x^2}right)}} \
& = lim_{xto-infty} {2x-1over -xsqrt{3+frac{1}{x}+frac{1}{x^2}}} \
& = lim_{xto-infty} {-2+frac{1}{x}over sqrt{3+frac{1}{x}+frac{1}{x^2}}} \
& = {-2oversqrt{3}}
end{align}$$



the second step is justified because $xto-infty$ implies $xlt0$, so $sqrt{x^2}=-x$.



For my attempt I ended up with the negative of the correct answer:



$$begin{align}
lim_{xto-infty} {2x-1over sqrt{3x^2+x+1}} & = lim_{xto-infty} left({2x-1over sqrt{3x^2+x+1}}cdotfrac{frac{1}{x}}{frac{1}{x}}right) \
& = lim_{xto-infty} {2-frac{1}{x}over sqrt{frac{1}{x^2}left(3x^2+x+1right)}} \
& = lim_{xto-infty} {2-frac{1}{x}over sqrt{3+frac{1}{x}+frac{1}{x^2}}} \
& = {2oversqrt{3}}
end{align}$$



Where have I gone wrong? I suspect the mistake lies in my second step, but I'm unable to identify what went wrong exactly.










share|cite|improve this question











$endgroup$








  • 18




    $begingroup$
    Also, thank you very much for including a full explanation of your thoughts, enough context to know what you can use, and a clear identification of where you think your error is. This is a well-written question.
    $endgroup$
    – T. Bongers
    Dec 11 '18 at 2:46








  • 2




    $begingroup$
    It is worth pointing out that we must always check the conditions before applying any 'rule'. Relevant to this question is the fact that for reals $a,b,c$ we have $(a^b)^c = a^{b·c}$ if $a$ is positive, and not necessarily so otherwise: $((-1)^6)^{1/2} ne (-1)^{6·1/2}$.
    $endgroup$
    – user21820
    Dec 11 '18 at 9:08










  • $begingroup$
    In the future, it may help to substitute $x$ with $-x$ when dealing with limits on the negative axis.
    $endgroup$
    – Gautam Shenoy
    Dec 11 '18 at 14:55






  • 1




    $begingroup$
    Note that $displaystyle,sqrt{,{a^{2}},}, = leftvert,{a},rightvert$
    $endgroup$
    – Felix Marin
    Dec 12 '18 at 5:33
















24












$begingroup$


My textbook asks me to evaluate the limit $$lim_{xto-infty} {2x-1over sqrt{3x^2+x+1}}$$ which evaluates to $-2oversqrt{3}$. The method in the book is to factor out $x^2$ from the root in the denominator:



$$begin{align}
lim_{xto-infty} {2x-1over sqrt{3x^2+x+1}} & = lim_{xto-infty} {2x-1over sqrt{x^2left(3+frac{1}{x}+frac{1}{x^2}right)}} \
& = lim_{xto-infty} {2x-1over -xsqrt{3+frac{1}{x}+frac{1}{x^2}}} \
& = lim_{xto-infty} {-2+frac{1}{x}over sqrt{3+frac{1}{x}+frac{1}{x^2}}} \
& = {-2oversqrt{3}}
end{align}$$



the second step is justified because $xto-infty$ implies $xlt0$, so $sqrt{x^2}=-x$.



For my attempt I ended up with the negative of the correct answer:



$$begin{align}
lim_{xto-infty} {2x-1over sqrt{3x^2+x+1}} & = lim_{xto-infty} left({2x-1over sqrt{3x^2+x+1}}cdotfrac{frac{1}{x}}{frac{1}{x}}right) \
& = lim_{xto-infty} {2-frac{1}{x}over sqrt{frac{1}{x^2}left(3x^2+x+1right)}} \
& = lim_{xto-infty} {2-frac{1}{x}over sqrt{3+frac{1}{x}+frac{1}{x^2}}} \
& = {2oversqrt{3}}
end{align}$$



Where have I gone wrong? I suspect the mistake lies in my second step, but I'm unable to identify what went wrong exactly.










share|cite|improve this question











$endgroup$








  • 18




    $begingroup$
    Also, thank you very much for including a full explanation of your thoughts, enough context to know what you can use, and a clear identification of where you think your error is. This is a well-written question.
    $endgroup$
    – T. Bongers
    Dec 11 '18 at 2:46








  • 2




    $begingroup$
    It is worth pointing out that we must always check the conditions before applying any 'rule'. Relevant to this question is the fact that for reals $a,b,c$ we have $(a^b)^c = a^{b·c}$ if $a$ is positive, and not necessarily so otherwise: $((-1)^6)^{1/2} ne (-1)^{6·1/2}$.
    $endgroup$
    – user21820
    Dec 11 '18 at 9:08










  • $begingroup$
    In the future, it may help to substitute $x$ with $-x$ when dealing with limits on the negative axis.
    $endgroup$
    – Gautam Shenoy
    Dec 11 '18 at 14:55






  • 1




    $begingroup$
    Note that $displaystyle,sqrt{,{a^{2}},}, = leftvert,{a},rightvert$
    $endgroup$
    – Felix Marin
    Dec 12 '18 at 5:33














24












24








24


3



$begingroup$


My textbook asks me to evaluate the limit $$lim_{xto-infty} {2x-1over sqrt{3x^2+x+1}}$$ which evaluates to $-2oversqrt{3}$. The method in the book is to factor out $x^2$ from the root in the denominator:



$$begin{align}
lim_{xto-infty} {2x-1over sqrt{3x^2+x+1}} & = lim_{xto-infty} {2x-1over sqrt{x^2left(3+frac{1}{x}+frac{1}{x^2}right)}} \
& = lim_{xto-infty} {2x-1over -xsqrt{3+frac{1}{x}+frac{1}{x^2}}} \
& = lim_{xto-infty} {-2+frac{1}{x}over sqrt{3+frac{1}{x}+frac{1}{x^2}}} \
& = {-2oversqrt{3}}
end{align}$$



the second step is justified because $xto-infty$ implies $xlt0$, so $sqrt{x^2}=-x$.



For my attempt I ended up with the negative of the correct answer:



$$begin{align}
lim_{xto-infty} {2x-1over sqrt{3x^2+x+1}} & = lim_{xto-infty} left({2x-1over sqrt{3x^2+x+1}}cdotfrac{frac{1}{x}}{frac{1}{x}}right) \
& = lim_{xto-infty} {2-frac{1}{x}over sqrt{frac{1}{x^2}left(3x^2+x+1right)}} \
& = lim_{xto-infty} {2-frac{1}{x}over sqrt{3+frac{1}{x}+frac{1}{x^2}}} \
& = {2oversqrt{3}}
end{align}$$



Where have I gone wrong? I suspect the mistake lies in my second step, but I'm unable to identify what went wrong exactly.










share|cite|improve this question











$endgroup$




My textbook asks me to evaluate the limit $$lim_{xto-infty} {2x-1over sqrt{3x^2+x+1}}$$ which evaluates to $-2oversqrt{3}$. The method in the book is to factor out $x^2$ from the root in the denominator:



$$begin{align}
lim_{xto-infty} {2x-1over sqrt{3x^2+x+1}} & = lim_{xto-infty} {2x-1over sqrt{x^2left(3+frac{1}{x}+frac{1}{x^2}right)}} \
& = lim_{xto-infty} {2x-1over -xsqrt{3+frac{1}{x}+frac{1}{x^2}}} \
& = lim_{xto-infty} {-2+frac{1}{x}over sqrt{3+frac{1}{x}+frac{1}{x^2}}} \
& = {-2oversqrt{3}}
end{align}$$



the second step is justified because $xto-infty$ implies $xlt0$, so $sqrt{x^2}=-x$.



For my attempt I ended up with the negative of the correct answer:



$$begin{align}
lim_{xto-infty} {2x-1over sqrt{3x^2+x+1}} & = lim_{xto-infty} left({2x-1over sqrt{3x^2+x+1}}cdotfrac{frac{1}{x}}{frac{1}{x}}right) \
& = lim_{xto-infty} {2-frac{1}{x}over sqrt{frac{1}{x^2}left(3x^2+x+1right)}} \
& = lim_{xto-infty} {2-frac{1}{x}over sqrt{3+frac{1}{x}+frac{1}{x^2}}} \
& = {2oversqrt{3}}
end{align}$$



Where have I gone wrong? I suspect the mistake lies in my second step, but I'm unable to identify what went wrong exactly.







calculus algebra-precalculus limits






share|cite|improve this question















share|cite|improve this question













share|cite|improve this question




share|cite|improve this question








edited Dec 12 '18 at 4:44







Cdizzle

















asked Dec 11 '18 at 2:41









CdizzleCdizzle

1636




1636








  • 18




    $begingroup$
    Also, thank you very much for including a full explanation of your thoughts, enough context to know what you can use, and a clear identification of where you think your error is. This is a well-written question.
    $endgroup$
    – T. Bongers
    Dec 11 '18 at 2:46








  • 2




    $begingroup$
    It is worth pointing out that we must always check the conditions before applying any 'rule'. Relevant to this question is the fact that for reals $a,b,c$ we have $(a^b)^c = a^{b·c}$ if $a$ is positive, and not necessarily so otherwise: $((-1)^6)^{1/2} ne (-1)^{6·1/2}$.
    $endgroup$
    – user21820
    Dec 11 '18 at 9:08










  • $begingroup$
    In the future, it may help to substitute $x$ with $-x$ when dealing with limits on the negative axis.
    $endgroup$
    – Gautam Shenoy
    Dec 11 '18 at 14:55






  • 1




    $begingroup$
    Note that $displaystyle,sqrt{,{a^{2}},}, = leftvert,{a},rightvert$
    $endgroup$
    – Felix Marin
    Dec 12 '18 at 5:33














  • 18




    $begingroup$
    Also, thank you very much for including a full explanation of your thoughts, enough context to know what you can use, and a clear identification of where you think your error is. This is a well-written question.
    $endgroup$
    – T. Bongers
    Dec 11 '18 at 2:46








  • 2




    $begingroup$
    It is worth pointing out that we must always check the conditions before applying any 'rule'. Relevant to this question is the fact that for reals $a,b,c$ we have $(a^b)^c = a^{b·c}$ if $a$ is positive, and not necessarily so otherwise: $((-1)^6)^{1/2} ne (-1)^{6·1/2}$.
    $endgroup$
    – user21820
    Dec 11 '18 at 9:08










  • $begingroup$
    In the future, it may help to substitute $x$ with $-x$ when dealing with limits on the negative axis.
    $endgroup$
    – Gautam Shenoy
    Dec 11 '18 at 14:55






  • 1




    $begingroup$
    Note that $displaystyle,sqrt{,{a^{2}},}, = leftvert,{a},rightvert$
    $endgroup$
    – Felix Marin
    Dec 12 '18 at 5:33








18




18




$begingroup$
Also, thank you very much for including a full explanation of your thoughts, enough context to know what you can use, and a clear identification of where you think your error is. This is a well-written question.
$endgroup$
– T. Bongers
Dec 11 '18 at 2:46






$begingroup$
Also, thank you very much for including a full explanation of your thoughts, enough context to know what you can use, and a clear identification of where you think your error is. This is a well-written question.
$endgroup$
– T. Bongers
Dec 11 '18 at 2:46






2




2




$begingroup$
It is worth pointing out that we must always check the conditions before applying any 'rule'. Relevant to this question is the fact that for reals $a,b,c$ we have $(a^b)^c = a^{b·c}$ if $a$ is positive, and not necessarily so otherwise: $((-1)^6)^{1/2} ne (-1)^{6·1/2}$.
$endgroup$
– user21820
Dec 11 '18 at 9:08




$begingroup$
It is worth pointing out that we must always check the conditions before applying any 'rule'. Relevant to this question is the fact that for reals $a,b,c$ we have $(a^b)^c = a^{b·c}$ if $a$ is positive, and not necessarily so otherwise: $((-1)^6)^{1/2} ne (-1)^{6·1/2}$.
$endgroup$
– user21820
Dec 11 '18 at 9:08












$begingroup$
In the future, it may help to substitute $x$ with $-x$ when dealing with limits on the negative axis.
$endgroup$
– Gautam Shenoy
Dec 11 '18 at 14:55




$begingroup$
In the future, it may help to substitute $x$ with $-x$ when dealing with limits on the negative axis.
$endgroup$
– Gautam Shenoy
Dec 11 '18 at 14:55




1




1




$begingroup$
Note that $displaystyle,sqrt{,{a^{2}},}, = leftvert,{a},rightvert$
$endgroup$
– Felix Marin
Dec 12 '18 at 5:33




$begingroup$
Note that $displaystyle,sqrt{,{a^{2}},}, = leftvert,{a},rightvert$
$endgroup$
– Felix Marin
Dec 12 '18 at 5:33










1 Answer
1






active

oldest

votes


















26












$begingroup$

Your mistake is in writing



$$frac 1 x = sqrt{frac{1}{x^2}}.$$



Since $x < 0$, the correct version includes a negative sign.






share|cite|improve this answer











$endgroup$













    Your Answer





    StackExchange.ifUsing("editor", function () {
    return StackExchange.using("mathjaxEditing", function () {
    StackExchange.MarkdownEditor.creationCallbacks.add(function (editor, postfix) {
    StackExchange.mathjaxEditing.prepareWmdForMathJax(editor, postfix, [["$", "$"], ["\\(","\\)"]]);
    });
    });
    }, "mathjax-editing");

    StackExchange.ready(function() {
    var channelOptions = {
    tags: "".split(" "),
    id: "69"
    };
    initTagRenderer("".split(" "), "".split(" "), channelOptions);

    StackExchange.using("externalEditor", function() {
    // Have to fire editor after snippets, if snippets enabled
    if (StackExchange.settings.snippets.snippetsEnabled) {
    StackExchange.using("snippets", function() {
    createEditor();
    });
    }
    else {
    createEditor();
    }
    });

    function createEditor() {
    StackExchange.prepareEditor({
    heartbeatType: 'answer',
    autoActivateHeartbeat: false,
    convertImagesToLinks: true,
    noModals: true,
    showLowRepImageUploadWarning: true,
    reputationToPostImages: 10,
    bindNavPrevention: true,
    postfix: "",
    imageUploader: {
    brandingHtml: "Powered by u003ca class="icon-imgur-white" href="https://imgur.com/"u003eu003c/au003e",
    contentPolicyHtml: "User contributions licensed under u003ca href="https://creativecommons.org/licenses/by-sa/3.0/"u003ecc by-sa 3.0 with attribution requiredu003c/au003e u003ca href="https://stackoverflow.com/legal/content-policy"u003e(content policy)u003c/au003e",
    allowUrls: true
    },
    noCode: true, onDemand: true,
    discardSelector: ".discard-answer"
    ,immediatelyShowMarkdownHelp:true
    });


    }
    });














    draft saved

    draft discarded


















    StackExchange.ready(
    function () {
    StackExchange.openid.initPostLogin('.new-post-login', 'https%3a%2f%2fmath.stackexchange.com%2fquestions%2f3034808%2fwhy-does-trying-to-compute-lim-x-to-infty-2x-1-over-sqrt3x2x1-resu%23new-answer', 'question_page');
    }
    );

    Post as a guest















    Required, but never shown

























    1 Answer
    1






    active

    oldest

    votes








    1 Answer
    1






    active

    oldest

    votes









    active

    oldest

    votes






    active

    oldest

    votes









    26












    $begingroup$

    Your mistake is in writing



    $$frac 1 x = sqrt{frac{1}{x^2}}.$$



    Since $x < 0$, the correct version includes a negative sign.






    share|cite|improve this answer











    $endgroup$


















      26












      $begingroup$

      Your mistake is in writing



      $$frac 1 x = sqrt{frac{1}{x^2}}.$$



      Since $x < 0$, the correct version includes a negative sign.






      share|cite|improve this answer











      $endgroup$
















        26












        26








        26





        $begingroup$

        Your mistake is in writing



        $$frac 1 x = sqrt{frac{1}{x^2}}.$$



        Since $x < 0$, the correct version includes a negative sign.






        share|cite|improve this answer











        $endgroup$



        Your mistake is in writing



        $$frac 1 x = sqrt{frac{1}{x^2}}.$$



        Since $x < 0$, the correct version includes a negative sign.







        share|cite|improve this answer














        share|cite|improve this answer



        share|cite|improve this answer








        answered Dec 11 '18 at 2:44


























        community wiki





        T. Bongers































            draft saved

            draft discarded




















































            Thanks for contributing an answer to Mathematics Stack Exchange!


            • Please be sure to answer the question. Provide details and share your research!

            But avoid



            • Asking for help, clarification, or responding to other answers.

            • Making statements based on opinion; back them up with references or personal experience.


            Use MathJax to format equations. MathJax reference.


            To learn more, see our tips on writing great answers.




            draft saved


            draft discarded














            StackExchange.ready(
            function () {
            StackExchange.openid.initPostLogin('.new-post-login', 'https%3a%2f%2fmath.stackexchange.com%2fquestions%2f3034808%2fwhy-does-trying-to-compute-lim-x-to-infty-2x-1-over-sqrt3x2x1-resu%23new-answer', 'question_page');
            }
            );

            Post as a guest















            Required, but never shown





















































            Required, but never shown














            Required, but never shown












            Required, but never shown







            Required, but never shown

































            Required, but never shown














            Required, but never shown












            Required, but never shown







            Required, but never shown







            Popular posts from this blog

            Quarter-circle Tiles

            build a pushdown automaton that recognizes the reverse language of a given pushdown automaton?

            Mont Emei